找回密码
 欢迎注册
楼主: kastin

[讨论] 特殊分式不等式

[复制链接]
发表于 2016-7-2 18:14:49 | 显示全部楼层
aimisiyou 发表于 2016-7-2 14:46
\(因为a,b,c>0且a+b+c=3,故其中至少有一数不小于1,不妨令其为a=1+t,t\in[0,2)\\2-t=b+c\geqslant 2\sqrt { ...


练习下不等式运算,\(\frac{1}{a}+\frac{1}{b}+\frac{1}{c}\ge\frac{(1+1+1)^2}{a+b+c}=3\)
毋因群疑而阻独见  毋任己意而废人言
毋私小惠而伤大体  毋借公论以快私情
发表于 2016-7-2 21:39:59 | 显示全部楼层
对于三个未知数全对称不等式,也可以利用三次方程来判别,比如说对于使用了变量a,b,c的全对称不等式,我们先看三次方程
$f(x)=(x-a)(x-b)(x-c)=x^3-\alpha x^2+\beta x-\gamma$
求导得到导函数为$f'(x)=3x^2-2\alpha x+\beta $,首先要求导函数有两个正根$x_1,x_2$,所以二次方程判别为大于0,不过这个就对应我们熟悉的不等式$\alpha^2>=3\beta$
于是设$f'(x)=0$两个根$x_1,x_2$,根据图像必然$f(x_1)<=0,f'(x_2)>=0$,所以$f(x_1)f(x_2)<=0$
由于$f(x)=(-2/9\alpha^2+2/3\beta)x+{\alpha\beta}/9-\gamma (mod f'(x))$,得出$f(x_1)f(x_2)=4/27*\gamma\alpha^3-1/27*\beta^2*\alpha^2-2/3*\alpha\beta\gamma+4/27*\beta^3+\gamma^2<=0$

$4\alpha^3\gamma+4\beta^3+27\gamma^2<=\alpha^2\beta^2+18\alpha\beta\gamma$
不过这个不等式好像过于复杂了

点评

这里有个软件包专门解决这种问题的http://www.is.pku.edu.cn//~xbc/software.html#intro  发表于 2016-7-4 19:41
多项式的模运算的系数应该是 $\alpha, \beta, \gamma$吧  发表于 2016-7-2 23:36
毋因群疑而阻独见  毋任己意而废人言
毋私小惠而伤大体  毋借公论以快私情
发表于 2016-7-2 22:02:18 来自手机 | 显示全部楼层
上面这个复杂不等式加上两个基本平均不等式应该构成了方程有三个正根的充要条件,也就是理论上它们可以推导出所有三正变量全对称不等式
毋因群疑而阻独见  毋任己意而废人言
毋私小惠而伤大体  毋借公论以快私情
发表于 2016-7-2 22:19:31 | 显示全部楼层
怎么证明\(abc \le 1\)?是根据\(abc\le (\frac{a+b+c}{3})^3\)吗?不等式公式全忘了……看来要恶补了。

点评

是的  发表于 2016-7-2 22:38
毋因群疑而阻独见  毋任己意而废人言
毋私小惠而伤大体  毋借公论以快私情
发表于 2016-7-3 11:17:20 | 显示全部楼层
mathe 发表于 2016-7-2 21:39
对于三个未知数全对称不等式,也可以利用三次方程来判别,比如说对于使用了变量a,b,c的全对称不等式,我们 ...


这个总结很有创意。我来练练手吧:
设三次方程为$ f(x)=(x-a)(x-b)(x-c)=x^3-\alpha x^2+\beta x-\gamma$,命名其维达定理衍生的各种式子为 基本式,即第零式。(同时隐藏关系:$\alpha>0,\beta>0,\gamma>0$)
命名导函数的判别式大于零为第一式:\[\alpha^2>=3\beta  \]
命名其两个拐点一正一负为第二式,  \[ 4\alpha^3\gamma+4\beta^3+27\gamma^2<=\alpha^2\beta^2+18\alpha\beta\gamma  \]

对于  $a^3+b^3+c^3>3abc$  只需应用基本式+第一式 得到 $a^3+b^3+c^3 -3abc=(\alpha ^3-3 \alpha  \beta +3 \gamma)-3 \gamma >=0$
对于本题 `\D\frac{1}{a^2}+\frac{1}{b^2}+\frac{1}{c^2}-(a^2+b^2+c^2)`,运用基本式,得到 \[\frac{1}{a^2}+\frac{1}{b^2}+\frac{1}{c^2}-(a^2+b^2+c^2)=\frac{-\alpha ^2 \gamma ^2-2 \alpha  \gamma +\beta ^2+2 \beta  \gamma ^2}{\gamma ^2}\]
额,好像看不到头绪,我还是直接交给软件吧,不过,软件好像也罢工了,还不如直接计算原表达效率高。


============================================
在此,我用的是Mathematica代码计算的,一条命令
RootSum[#^3 - \[Alpha] #^2 + \[Beta] # - \[Gamma] &, 1/#^2 - #^2 &]

点评

试试这个函数 SymmetricReduction  发表于 2016-7-3 20:34
毋因群疑而阻独见  毋任己意而废人言
毋私小惠而伤大体  毋借公论以快私情
发表于 2016-7-3 13:19:07 | 显示全部楼层
$4\alpha^3\gamma+4\beta^3+27\gamma^2<=\alpha^2\beta^2+18\alpha\beta\gamma$
看成关于$\gamma$的二次方程变为
$27\gamma^2+(4\alpha^3-18\alpha\beta)\gamma+4\beta^3-\alpha^2\beta^2<=0$
非常有意思的是这个二次方程根的判别式为$16(\alpha^2-3\beta)^3$,由此我们可以得出一个丑陋的公式
${9\alpha\beta-2\alpha^3-2(\alpha^2-3\beta)^{3/2}}/27<=\gamma<={9\alpha\beta-2\alpha^3+2(\alpha^2-3\beta)^{3/2}}/27$
如果我们记$h=\sqrt{\alpha^2-3beta}$,于是可以得出
$3\alpha\beta-2h^2(\alpha+h)<=27\gamma<=3\alpha\beta-2h^2(\alpha-h)$

点评

奇怪了,我怎么得不到这个判别式的三次方的表达  发表于 2016-7-3 14:37
哈哈,我就不去尝试了, ^_^  发表于 2016-7-3 13:36
毋因群疑而阻独见  毋任己意而废人言
毋私小惠而伤大体  毋借公论以快私情
发表于 2016-7-3 13:46:15 | 显示全部楼层
wanye在15#的不等式的问题是还没有齐次化,分子应该改为$-\alpha ^2 \gamma ^2+2 \beta  \gamma ^2+(-2 \alpha  \gamma +\beta ^2)(\alpha/3)^4$
于是要求证明$(\alpha ^2-2\beta)\gamma ^2+2/81 \alpha^5  \gamma -1/81 \beta ^2\alpha^4<=0$
显然二次项系数大于0而且方程两根互为相反数,所以只要证明对于$\gamma=3\alpha\beta-2h^2(\alpha-h)$时不等式成立即可
然后我们将$\beta={\alpha^2-h^2}/3$代入,得出一个不等式
$8h^8-24\alpha h^7+22\alpha^2h^6-4\alpha^3h^5-6\alpha^4h^4+8\alpha^5h^3-4\alpha^6h^2<=0$
于是可以写成
$h^2(h-\alpha)^2(4h^4-4h^3\alpha-h^2\alpha^2-2\alpha^4)<=0$
这个相当于要求$h<=1.374...\alpha$或$\h^2<=1.889...\alpha^2$
或者说$a^2+b^2+c^2-ab-bc-ca<=1.889...(a^2+b^2+c^2)$这是显然的

点评

次数标准没弄错。乘以$\frac{\alpha}{3}$没搞明白,现在懂了。原来是借用了$\frac{\alpha}{3}=1$了啊  发表于 2016-7-3 18:06
次数是以$a,b,c$的次数为标准,而不是$\alpha,\beta,\gamma$的次数  发表于 2016-7-3 15:57
次数不够的就乘上$\alpha/3$  发表于 2016-7-3 15:56
齐次化怎么做的  发表于 2016-7-3 14:25
毋因群疑而阻独见  毋任己意而废人言
毋私小惠而伤大体  毋借公论以快私情
发表于 2016-7-3 13:51:19 | 显示全部楼层
不等式$3\alpha\beta-2h^2(\alpha+h)<=27\gamma<=3\alpha\beta-2h^2(\alpha-h)$中
直接将$3\beta=\alpha^2-h^2$替换以后看起来会更加舒服
也就是
$\alpha(\alpha^2-h^2)-2h^2(\alpha+h)<=27\gamma<=\alpha(\alpha^2-h^2)-2h^2(\alpha-h)$
$(\alpha+h)(\alpha^2-h\alpha-2h^2)<=27\gamma<=(\alpha-h)(\alpha^2+h\alpha-h^2)$
毋因群疑而阻独见  毋任己意而废人言
毋私小惠而伤大体  毋借公论以快私情
 楼主| 发表于 2016-7-3 15:10:59 | 显示全部楼层
这道问题因为结构形式太特殊,基本上不太容易用基本不等式证明。在好几本书上搜寻过这道问题的答案,证明方法挺多的,只是都比较复杂,没有一个令人满意的简单方法。限于篇幅,这里简单介绍一下思路。

证法1:万能的差分配方法(SOS,Sum of Squares)
左右相减,找到分子形如 `S_a(b-c)^2+S_b(c-a)^2+S_c(a-b)^2`的表达形式,其中 `S_a,S_b,S_c` 为 `a,b,c` 的多项式,尽量是正号为好(有负号也不怕),然后分析得到大于等于零的结论。

证法2:分离变量利用函数(还是这种简单一些)
考虑 `\D f(x)=\frac{1}{x^2}-x^2+4x-4 \quad (0 < x < 3)`,注意到$$f(x)=\frac{(x-1)^2(1+\sqrt{2}-x)(x-1+\sqrt{2})}{x^2}$$因此 `x\in(0,1+\sqrt{2}]` 时,`f(x)\geqslant 0`. 显然若 `0< a,b,c,d\leqslant 1+\sqrt{2}(\approx 2.4)`时,原不等式成立。否则,`a,b,c` 中只能有一个超出该区间,不妨设是 `a` 大于2.4.
当 `1+\sqrt{2} < a < 3` 时,有 `b+c=3-a < 2-\sqrt{2}`,于是根据赫德尔不等式有$$\frac{1}{b^2}+\frac{1}{c^2}\geqslant \frac{(1+1)^3}{(b+c)^2} > 23$$然而$$a^2+b^2+c^2 < a^2+(3-a)^2 = 2a(a-3)+9<9$$故显然原不等式成立。

证法3:递推法
先证明2元情况,然后通过换元`\D x=\frac{2a}{3-c},y=\frac{2b}{3-c}` 于是 `x+y=2`,降低3元为2元情形。然后对 `c` 的范围进行分类讨论就能得到。这种方法能递推证明 `n` 个变量情形。目前只知道 `2\leq n \leq 9\;(n\in \NN^+,a_i > 0)`,`\D\sum_{i=1}^{n}a_i=n` 时,有 $$\D\frac{1}{a_1^2}+\frac{1}{a_2^2}+\frac{1}{a_3^2}+\cdots+\frac{1}{a_n^2}\geq a_1^2+a_2^2+a_3^2+\cdots+a_n^2$$

点评

就是函数图像是一段下凸和一段上凸情况,那么根据琴生不等式,取最小值时,所有下凸区间的点重叠,而上凸区间,除去边界外,上凸区间最多一个点。http://bbs.emath.ac.cn/thread-164-1-1.html 中应该使用过  发表于 2016-7-4 13:47
@mathe,总感觉这种类型的问题应该有一种统一的简便证法,就如你提到的极值一部分相等的情况,已经出现过好多次了,是否可以提炼成一种一般性的结论?  发表于 2016-7-4 12:31
而$n=10$时有解,但是对应极值点处不等式还是满足。但是$n>=11$时就不行了  发表于 2016-7-4 06:58
利用函数的凹凸性,可以得出取极值时n-1个数相同,而且另外一个数处导数和这个也相同,得出两个数满足$a^3b^3=a^2+ab+b^2$,然后用约束$(n-1)a+b=n$得出在$2<=n<=9$时无解,所以最值只能n个数相等时取到。  发表于 2016-7-4 06:58
n=10也成立,但是n>=11就不成立了  发表于 2016-7-4 06:30
毋因群疑而阻独见  毋任己意而废人言
毋私小惠而伤大体  毋借公论以快私情
发表于 2016-7-3 18:31:38 | 显示全部楼层
关于齐次化。根本原因还是目标式子是非齐次的,我宁愿一开始就在目标式子这边下手。于是原题的真实面目是这样的(已经有Mathematica软件证实):
----------------
已知 $a,b,c>0$,求证 $(\frac{a+b+c}{3})^4(\frac{1}{a^2}+\frac{1}{b^2}+\frac{1}{c^2}) >=a^2+b^2+c^2$
--------------

我们可以大胆一点,可以推测 已知 $a,b,c>0$,$m>=n$是自然数, 求证 $(\frac{a+b+c}{3})^{m+n}(\frac{1}{a^m}+\frac{1}{b^m}+\frac{1}{c^m}) >=a^n+b^n+c^n$

甚至可以再大胆一点:
用mathe提及的赫尔德不等式表达,$M_k = ({x_1^k+x_2^k+...+x_n^k}/n)^{1/k}$, 将变量个数推广至任意个。

点评

wanye的这个猜想必然不成立,我们可以让m=n并保持a+b+c=3,两边同时开n次方让后让n趋向无穷得出max{1/a,1/b,1/c}>=max{a,b,c},这个显然是不是必须的  发表于 2016-7-4 06:21
毋因群疑而阻独见  毋任己意而废人言
毋私小惠而伤大体  毋借公论以快私情
您需要登录后才可以回帖 登录 | 欢迎注册

本版积分规则

小黑屋|手机版|数学研发网 ( 苏ICP备07505100号 )

GMT+8, 2024-4-20 12:08 , Processed in 0.068414 second(s), 15 queries .

Powered by Discuz! X3.5

© 2001-2024 Discuz! Team.

快速回复 返回顶部 返回列表